2006 AMC 12A Problems/Problem 12

Revision as of 19:41, 31 January 2007 by Azjps (talk | contribs) (Solution: should use show preview button more often... anyway, more typos)

Problem


An image is supposed to go here. You can help us out by creating one and editing it in. Thanks.


A number of linked rings, each 1 cm thick, are hanging on a peg. The top ring has an outside diameter of 20 cm. The outside diameter of each of the outer rings is 1 cm less than that of the ring above it. The bottom ring has an outside diameter of 3 cm. What is the distance, in cm, from the top of the top ring to the bottom of the bottom ring?

$\mathrm{(A) \ } 171\qquad \mathrm{(B) \ } 173\qquad \mathrm{(C) \ } 182\qquad \mathrm{(D) \ } 188$$\mathrm{(E) \ }  210$

Solution

The sum of the consecutively increasing integers from 3 to 20 is $\frac{1}{2}(18)(3+20) = 207$. However, the 17 intersections between the rings must also be subtracted, so we get $207 - 2(17) = 173 \Rightarrow B$.

See also


{{{header}}}
Preceded by
Problem 11
AMC 12A
2006
Followed by
Problem 13